// Numbas version: exam_results_page_options {"name": "Maximum/minimum", "extensions": [], "custom_part_types": [], "resources": [["question-resources/Untitled.jpg", "/srv/numbas/media/question-resources/Untitled.jpg"], ["question-resources/Untitled_qCawkyB.jpg", "/srv/numbas/media/question-resources/Untitled_qCawkyB.jpg"]], "navigation": {"allowregen": true, "showfrontpage": false, "preventleave": false, "typeendtoleave": false}, "question_groups": [{"pickingStrategy": "all-ordered", "questions": [{"functions": {}, "ungrouped_variables": ["l", "w"], "name": "Maximum/minimum", "tags": [], "preamble": {"css": "", "js": ""}, "advice": "

The length of the box is \\(\\var{l}-2x\\), the width is \\(\\var{w}-2x\\) and the height is \\(x\\).

\n

The volume is then given by

\n

\\(V=(\\var{l}-2x).(\\var{w}-2x).x\\)

\n

\\(V=\\simplify{4x^3-2*({w}+{l})x^2+{l}*{w}x}\\)

\n

\\(\\frac{dV}{dx}=\\simplify{12x^2-4*({w}+{l})x+{l}*{w}}\\)

\n

This is a quadratic equation.

\n

\\(x=\\frac{\\simplify{4*({w}+{l})}\\pm\\sqrt(\\simplify{16*({w}+{l})^2-48*{w}*{l}})}{24}\\)

\n

\\(x=\\frac{\\simplify{{w}+{l}}\\pm\\sqrt(\\simplify{{w}^2-{w}*{l}+{l}^2})}{6}\\)

\n

\\(\\frac{d^2V}{dx^2}=\\simplify{24x-4*({w}+{l})}\\)

\n

when  \\(x=\\simplify{({w}+{l}-sqrt({w}^2-{w}*{l}+{l}^2))/6}\\)           \\(\\frac{d^2V}{dx^2}<0\\)      and therefore is the value that gives a maximum.

", "rulesets": {}, "parts": [{"prompt": "

Input the value for \\(x\\) correct to one decimal place.

\n

\\(x = \\) [[0]]

", "variableReplacements": [], "variableReplacementStrategy": "originalfirst", "gaps": [{"precisionType": "dp", "precisionMessage": "You have not given your answer to the correct precision.", "allowFractions": false, "variableReplacements": [], "maxValue": "({w}+{l}-sqrt({w}^2+{l}^2-{w}*{l}))/6", "strictPrecision": false, "minValue": "({w}+{l}-sqrt({w}^2+{l}^2-{w}*{l}))/6", "variableReplacementStrategy": "originalfirst", "precisionPartialCredit": 0, "correctAnswerFraction": false, "showCorrectAnswer": true, "precision": "1", "scripts": {}, "marks": 1, "showPrecisionHint": false, "type": "numberentry"}], "showCorrectAnswer": true, "scripts": {}, "marks": 0, "type": "gapfill"}], "extensions": [], "statement": "

A rectangular sheet of metal of length = \\(\\var{l}cm\\) and width  = \\(\\var{w}cm\\) has a square of side \\(x\\,cm\\) cut from each corner. The ends and sides will be folded upwards to form an open box.

\n

Determine the value of \\(x\\) that will maximise the volume of this box.

\n

", "variable_groups": [], "variablesTest": {"maxRuns": 100, "condition": ""}, "variables": {"l": {"definition": "random(30..100#5)", "templateType": "randrange", "group": "Ungrouped variables", "name": "l", "description": ""}, "w": {"definition": "random(10..28#2)", "templateType": "randrange", "group": "Ungrouped variables", "name": "w", "description": ""}}, "metadata": {"description": "

Maximising the volume of a rectangular box

", "licence": "Creative Commons Attribution 4.0 International"}, "type": "question", "contributors": [{"name": "Frank Doheny", "profile_url": "https://numbas.mathcentre.ac.uk/accounts/profile/789/"}]}]}], "contributors": [{"name": "Frank Doheny", "profile_url": "https://numbas.mathcentre.ac.uk/accounts/profile/789/"}]}